数学分析 - Eufisky - The lost book

数学系自学视频课程

1.数学分析,王昆扬,北京师范大学,http://video.chaoxing.com/play_400001575_18227.shtml

2.高等代数,丘维生,北京大学,http://video.chaoxing.com/serie_400015565.shtml

3.高等代数,李尚志,北京航天航空大学,http://video.chaoxing.com/serie_400001055.shtml

4.实变函数,上海交大,http://video.1kejian.com/university/ggkc/12481/或者http://www.dxzy163.com/view/index6750.html后者包括一些泛函分析

5.抽象代数,邓少强,南开大学http://www.icourses.cn/jpk/viewCharacterDetail.action?courseId=5968§ionId=100992

6.抽象代数,章璞,上海交大,http://video.chaoxing.com/serie_400008049.shtml

7.泛函分析.国立台湾大学,http://video.1kejian.com/video/?68356-0-0.html

8.高等代数,杜现昆,吉林大学,http://www.icourses.cn/coursestatic/course_3399.html

9.常微分方程,史少云,吉林大学,http://www.icourses.cn/coursestatic/course_6531.html

10.数学物理方法,吴崇试,北京大学,http://www.icourses.cn/coursestatic/course_3569.html

11.偏微分方程,朱长江,华中师范大学,http://www.icourses.cn/coursestatic/course_4147.html

12.复变函数,刘太顺,湖州师范学院,http://www.icourses.cn/coursestatic/course_3950.html

13.泛函分析,孙炯,内蒙古大学,http://www.icourses.cn/coursestatic/course_7021.html

14.高等代数,林亚南,厦门大学,http://www.icourses.cn/coursestatic/course_3077.html

国立交通大学开放式课程:http://ocw.nctu.edu.tw/index.php

15.台湾大学陈金次老师高等微积分http://ocw.aca.ntu.edu.tw/ntu-ocw/index.php/ocw/cou/101S130/2/V/1?v=ntu

豆瓣视频推荐http://www.douban.com/group/topic/50463538/

善科数学视频:http://www.mysanco.cn/index.php?class=video&action=video_collection#tab9

爱课程网http://www.icourses.cn/home/

若干个著名的积分及文献

1.Ising Integrals
 
1.Integrals of the Ising class (D.H. Bailey J.M. Borwein R.E.Crandall)

2.Hypergeometric forms for Ising-class integrals (D.H. Bailey, D. Borwein, J.M. Borwein,R.E. Crandall)

3. Finding General Explicit Formulas for Ising Integral Recursions (D.H. Bailey J.M. Borwein)

4.On Recurrences for Ising Integrals (Johannes Kepler University Linz, Austria)
2.watson Integrals
 
1.THREE TRIPLE INTEGRALS (G. N. WATSON)

2.WATSON'S THIRD INTEGRAL (Hannah Cairns)

3.ON THE EVALUATION OF GENERALIZED WATSON INTEGRALS (G. S. JOYCE AND I. J. ZUCKER)
\begin{align*}&{I_1} = \frac{1}{{{\pi ^3}}}\int_0^\pi  {\int_0^\pi  {\int_0^\pi  {\frac{{dudvdw}}{{1 - \cos u\cos v\cos w}}} } }  = \frac{{4{{\left[ {K\left( {\frac{1}{2}\sqrt 2 } \right)} \right]}^2}}}{{{\pi ^2}}} = \frac{{{\Gamma ^4}\left( {\frac{1}{4}} \right)}}{{4{\pi ^3}}}\\&{I_2} = \frac{1}{{{\pi ^3}}}\int_0^\pi  {\int_0^\pi  {\int_0^\pi  {\frac{{dudvdw}}{{3 - \cos u\cos v - \cos w\cos u - \cos u\cos v}}} } } \\&= \frac{{\sqrt 3 {{\left[ {K\left( {\frac{1}{4}\left( {\sqrt 6  - \sqrt 2 } \right)} \right)} \right]}^2}}}{{{\pi ^2}}} = \frac{{3{\Gamma ^6}\left( {\frac{1}{3}} \right)}}{{{2^{14/3}}{\pi ^4}}}\\&{I_3} = \frac{1}{{{\pi ^3}}}\int_0^\pi  {\int_0^\pi  {\int_0^\pi  {\frac{{dudvdw}}{{3 - \cos u - \cos v - \cos w}}} } } \\&= \frac{{4\left( {18 + 12\sqrt 2  - 10\sqrt 3  - 7\sqrt 6 } \right){{\left[ {K\left( {\left( {2 - \sqrt 3 } \right)\left( {\sqrt 3  - \sqrt 2 } \right)} \right)} \right]}^2}}}{{{\pi ^2}}}\\&= \frac{{\sqrt 6 }}{{96{\pi ^3}}}\Gamma \left( {\frac{1}{{24}}} \right)\Gamma \left( {\frac{5}{{24}}} \right)\Gamma \left( {\frac{7}{{24}}} \right)\Gamma \left( {\frac{{11}}{{24}}} \right).\end{align*}
3.Box Integrals
 
1.Box integrals (D.H. Bailey J.M. Borwein R.E. Crandall)

2.Higher-dimensional box integrals (Jonathan M. Borwein O-Yeat Chan y R. E. Crandall)

3.ADVANCES IN THE THEORY OF BOX INTEGRALS (D. H. BAILEY, J. M. BORWEIN, AND R. E. CRANDALL)
 
4.spin integrals
 
5.Lattice Sum
 
6.Euler Sum
 
1.Euler Sums and Contour Integral Representations (Philippe Flajolet and Bruno Salvy)

2.Experimental evaluation of Euler sums (D.H.Bailey J.M.Borwein andR.Girgensohn)

3.Evaluation of triple euler sums (Jonathan M. Borwein)

4.Harmonic sums,Mellin transforms and Integrals (J.A.M.Vermaseren,NIKHEF)
 
7.Meijer G function
 
8.Ramanujan-type series
 
9.PSLQ
 
来源:百度贴吧业余数学研究吧http://tieba.baidu.com/f?kw=%D2%B5%D3%E0%CA%FD%D1%A7%D1%D0%BE%BF

MatheMaticas中的巧妙范例[转载自哆嗒数学平台吧chzhn]

1.EllipticK 范例

在三维立点阵中随机访问并返回原点的概率:

1 - \[Pi]^2/

72 (6 + 2 Sqrt[3] + Sqrt[6]) EllipticK[

35 + 24 Sqrt[2] - 20 Sqrt[3] - 14 Sqrt[6]]^-2 // N

测试程序

BlockRandom[SeedRandom[11]; 
Count[Table[walkerPosition = {0, 0, 0}; steps = 0; 
While[steps == 0 || (steps < 100 && walkerPosition =!= {0, 0, 0}), 
steps++; 
walkerPosition = 
walkerPosition + {{1, 0, 0}, {-1, 0, 0}, {0, 1, 0}, {0, -1, 
0}, {0, 0, 1}, {0, 0, -1}}[[Random[Integer, {1, 6}]]]]; 
steps, {1000}], _?(# < 100 &)]]

2.Beta

贝塔函数倒数的$n\times n$ 矩阵的行列式为$n!$:

\[\left| {\begin{array}{*{20}{c}}{\frac{1}{{B\left( {1,1} \right)}}}&{\frac{1}{{B\left( {1,2} \right)}}}& \cdots &{\frac{1}{{B\left( {1,n} \right)}}}\\{\frac{1}{{B\left( {2,1} \right)}}}&{\frac{1}{{B\left( {2,2} \right)}}}& \cdots &{\frac{1}{{B\left( {2,n}\right)}}}\\{\frac{1}{{B\left( {3,1} \right)}}}&{\frac{1}{{B\left( {3,2} \right)}}}& \cdots &{\frac{1}{{B\left( {3,n} \right)}}}\\\cdots & \cdots & \cdots & \cdots \\{\frac{1}{{B\left( {n,1} \right)}}}&{\frac{1}{{B\left( {n,2} \right)}}}& \cdots &{\frac{1}{{B\left( {n,n} \right)}}}\end{array}} \right| = n!.\]

3.Binomial

希尔伯特矩阵的逆:\[{H_{ij}} = \frac{1}{{i + j - 1}}.\]

逆矩阵系数为

\[{\left( {{H^{ - 1}}} \right)_{ij}} = {\left( { - 1} \right)^{i + j}}\left( {i + j - 1} \right)\left( \begin{array}{l}n + i - 1\\n - j\end{array} \right)\left( \begin{array}{l}n + j - 1\\n - i\end{array} \right){\left( \begin{array}{l}i + j - 2\\i - 1\end{array} \right)^2}.\]

4.Erf

\[\frac{1}{{1 + \frac{1}{{1 + \frac{2}{{1 + \frac{3}{{1 + \frac{4}{{1 + \frac{5}{{1 + \frac{6}{{1 + \frac{7}{{1 + \frac{8}{{1 + \frac{9}{{1 +  \cdots }}}}}}}}}}}}}}}}}}}} = \sqrt {\frac{{\pi e}}{2}} \left( {1 - \rm{Erf}\left( {\frac{1}{{\sqrt 2 }}} \right)} \right).\]

5.HermiteH

广义 Lissajous 图形:

Block[{n = 11, m = 13},

ParametricPlot[{ Exp[-x^2/2] HermiteH[n, x]/Sqrt[2^n n!],

Exp[-x^2/2] HermiteH[m, x]/Sqrt[2^m m!]}, {x, -8, 8}]]

 

6.BesselI

等差数列的连分数表示

\[1 + \frac{1}{{2 + \frac{1}{{3 + \frac{1}{{4 + \frac{1}{{5 + \frac{1}{{6 +  \cdots }}}}}}}}}} = \frac{{{I_0}\left( 2 \right)}}{{{I_1}\left( 2 \right)}}.\]

代码ContinuedFraction[BesselI[0, 2]/BesselI[1, 2], 20]

输出

{1, 2, 3, 4, 5, 6, 7, 8, 9, 10, 11, 12, 13, 14, 15, 16, 17, 18, 19, 20}

7.Floor

数列$\{1, 2, 2, 3, 3, 3, 4, 4, 4, 4, 5, 5, 5, 5, 5, 6, 6, 6, 6, 6, 6\cdots\}的通项

\[{a_n} = \left[ {\sqrt {2k}  + \frac{1}{2}} \right].\]

8.Exp

(1)Exp迭代分形

DensityPlot[

Length @FixedPointList[

If[TrueQ[Abs[#] > 10.^5], Indeterminate, Exp[#/(x + I y)]] &,

x + I y, 10], {x, -1, 3}, {y, -1, 1}, MaxRecursion -> 4]

(2)黎曼- 维尔斯特拉斯函数任何位置不可微:

ParametricPlot[{Re[#], Im[#]} &@\!\(

\*UnderoverscriptBox[\(\[Sum]\), \(j = 1\), \(200\)]

\*FractionBox[

SuperscriptBox[\(E\), \(I\

\*SuperscriptBox[\(j\), \(3\)]\ \[CurlyPhi]\)],

SuperscriptBox[\(j\), \(2\)]]\), {\[CurlyPhi], 0, 2 \[Pi]}]

9.Tan

\[1 + \frac{1}{{1 + \frac{1}{{1 + \frac{1}{{3 + \frac{1}{{1 + \frac{1}{{5 + \frac{1}{{1 + \frac{1}{{7 +  \cdots }}}}}}}}}}}}}} = \tan 1.\]
10.Tanh
\[\frac{1}{{1 + \frac{1}{{3 + \frac{1}{{5 + \frac{1}{{7 + \frac{1}{{9 + \frac{1}{{11 + \frac{1}{{13 +  \cdots }}}}}}}}}}}}}} = \tanh 1.\]
11.sinc
\[\int_0^\infty  {\left( {\prod\limits_{k = 0}^n {\rm{sinc}\left( {\frac{x}{{2k + 1}}} \right)} } \right)dx}  = \pi \left( {n = 0,1,2,3,4,5,6} \right)\]

12.Factor

$x^n-1$在整数范围内充分分解后,几乎所有因式的系数不是1就是-1,但是也有一些例外,第一个例子是

\begin{align*}&{x^{105}} - 1 = ( - 1 + x)(1 + x + {x^2})(1 + x + {x^2} + {x^3} + {x^4})(1 + x + {x^2} + {x^3} + {x^4} + {x^5} + {x^6})\\&(1 - x + {x^3} - {x^4} + {x^5} - {x^7} + {x^8})(1 - x + {x^3} - {x^4} + {x^6} - {x^8} + {x^9} - x^{11} + x^{12})\\&(1 - x + {x^5} - {x^6} + {x^7} - {x^8} + x^{10} - {x^{11}} + {x^{12}} - {x^{13}} + {x^{14}} - {x^{16}} + {x^{17}} - {x^{18}} + {x^{19}} - {x^{23}} + {x^{24}})\\&(1 + x + {x^2} - {x^5} - {x^6} - 2{x^7} - {x^8} - {x^9} + {x^{12}} + {x^{13}} + {x^{14}} + {x^{15}} + {x^{16}} + {x^{17}} - {x^{20}} - {x^{22}} - {x^{24}} \\&- {x^{26}} - {x^{28}} + {x^{31}} + {x^{32}} + {x^{33}} + {x^{34}} + {x^{35}} + {x^{36}} - {x^{39}} - {x^{40}} - 2{x^{41}} - {x^{42}} - {x^{43}} + {x^{46}} + {x^{47}} + {x^{48}}).\end{align*}
参考来源:http://tieba.baidu.com/p/2978798364

逆神的数学分析题答案总算补全了

9月5号逆神在数学竞赛交流群里给了一份试题,建模结束后自己才真正仔细思考起来,经过各位大神的指教,终于能够把所有试题的答案给补全,难免存在错误,联系2609480070@qq.cm进行纠正.

数学分析练习题

来源:逆蝶

整理:1729

2014年9月5日

 

 

下面的习题均来自大学生数学竞赛群群友逆蝶提供的数学分析练习题,难免存在错误,请联系(2609480070)进行纠正!

1.已知$a_1=a_2=1,a_{n+2}=2a_{n+1}+3a_n,n=1,2,\ldots$,求幂级数$\sum\limits_{n=1}^\infty{a_n x^n}$的收敛半径,收敛域以及和函数.

解.\begin{align*}&{a_{n + 2}} + {a_{n + 1}} = 3\left( {{a_{n + 1}} + {a_n}} \right) \\\Rightarrow &{a_{n + 1}} + {a_n} = \left( {{a_2} + {a_1}} \right) \times {3^{n - 1}} = 2 \times {3^{n - 1}}\\&{a_{n + 1}} - \frac{1}{2} \times {3^n} = \left( { - 1} \right)\left( {{a_n} - \frac{1}{2} \times {3^{n - 1}}} \right) \\\Rightarrow &{a_n} - \frac{1}{2} \times {3^{n - 1}} = \frac{1}{2}{\left( { - 1} \right)^{n - 1}}.\end{align*}

\begin{align*}\sum\limits_{n = 1}^\infty {{a_n}{x^n}} &= \frac{1}{2}\sum\limits_{n = 1}^\infty {\left( {{3^{n - 1}} + {{\left( { - 1} \right)}^{n - 1}}} \right){x^n}} \\&= \frac{1}{6}\sum\limits_{n = 1}^\infty {{{\left( {3x} \right)}^n}} - \frac{1}{2}\sum\limits_{n = 1}^\infty {{{\left( { - x} \right)}^n}} \\&= \frac{1}{2}\frac{x}{{1 - 3x}} + \frac{1}{2}\frac{x}{{1 + x}} = \frac{{x\left( {1 - x} \right)}}{{\left( {1 + x} \right)\left( {1 - 3x} \right)}}.\end{align*}

由\[\mathop {\lim }\limits_{n \to \infty } \frac{{{a_{n + 1}}}}{{{a_n}}} = \mathop {\lim }\limits_{n \to \infty } \frac{{{3^n} + {{\left( { - 1} \right)}^n}}}{{{3^{n - 1}} + {{\left( { - 1} \right)}^{n - 1}}}} = 3\]

知幂级数收敛半径为$\frac13$,收敛域为$\left(-\frac13,\frac13\right)$.

 

2.计算级数\[\sum\limits_{n = 1}^\infty {\frac{1}{{n\left( {n + 1} \right)}}\left( {1 + \frac{1}{2} + \cdots + \frac{1}{n}} \right)} \]的和.

解.由Stolz公式

\[\mathop {\lim }\limits_{n \to \infty } \frac{{1 + \frac{1}{2} + \cdots + \frac{1}{{n + 1}}}}{{n + 1}} = \mathop {\lim }\limits_{n \to \infty } \frac{1}{{n + 2}} = 0.\]

我们有

\begin{align*}&\sum\limits_{n = 1}^\infty {\frac{1}{{n\left( {n + 1} \right)}}\left( {1 + \frac{1}{2} + \cdots + \frac{1}{n}} \right)} \\&= \sum\limits_{n = 1}^\infty {\frac{1}{{n\left( {n + 1} \right)}}\left[ {\left( {n + 1} \right)\left( {1 + \frac{1}{2} + \cdots + \frac{1}{n}} \right) - n\left( {1 + \frac{1}{2} + \cdots + \frac{1}{n} + \frac{1}{{n + 1}}} \right) + \frac{n}{{n + 1}}} \right]} \\&= \sum\limits_{n = 1}^\infty {\left( {\frac{{1 + \frac{1}{2} + \cdots + \frac{1}{n}}}{n} - \frac{{1 + \frac{1}{2} + \cdots + \frac{1}{{n + 1}}}}{{n + 1}}} \right)} + \sum\limits_{n = 1}^\infty {\frac{1}{{{{\left( {n + 1} \right)}^2}}}} \\&= 1 - \mathop {\lim }\limits_{n \to \infty } \frac{{1 + \frac{1}{2} + \cdots + \frac{1}{{n + 1}}}}{{n + 1}} + \frac{{{\pi ^2}}}{6} - 1 = \frac{{{\pi ^2}}}{6}.\end{align*}

3.设$\alpha$是实数,计算\[\int_0^\infty {\frac{{dx}}{{\left( {1 + {x^2}} \right)\left( {1 + {x^\alpha }} \right)}}} .\]

解.令$x=\tan t$,我们知

\begin{align*}\int_0^\infty {\frac{{dx}}{{\left( {1 + {x^2}} \right)\left( {1 + {x^\alpha }} \right)}}} &= \int_0^{\frac{\pi }{2}} {\frac{{{{\cos }^\alpha }t}}{{{{\sin }^\alpha }t + {{\cos }^\alpha }t}}dt} = \int_0^{\frac{\pi }{2}} {\frac{{{{\sin }^\alpha }t}}{{{{\sin }^\alpha }t + {{\cos }^\alpha }t}}dt} \\&= \frac{1}{2}\left( {\int_0^{\frac{\pi }{2}} {\frac{{{{\cos }^\alpha }t}}{{{{\sin }^\alpha }t + {{\cos }^\alpha }t}}dt} + \int_0^{\frac{\pi }{2}} {\frac{{{{\sin }^\alpha }t}}{{{{\sin }^\alpha }t + {{\cos }^\alpha }t}}dt} } \right)\\&= \frac{\pi }{4}.\end{align*}

4.设$f(x)$在$[0,\pi]$连续,求证:不能同时有\[\int_0^\pi {{{\left| {f\left( x \right) - \sin x} \right|}^2}dx} < \frac{\pi }{4},\int_0^\pi {{{\left| {f\left( x \right) - \cos x} \right|}^2}dx} < \frac{\pi }{4}.\]又问何时上面的两个不等式成为等式?

证.注意到

\[a^2+b^2\geq \frac{(a-b)^2}{2}.\]

我们有

\[\int_0^\pi {{{\left| {f\left( x \right) - \sin x} \right|}^2}dx} + \int_0^\pi {{{\left| {f\left( x \right) - \cos x} \right|}^2}dx} \ge \int_0^\pi {\frac{{{{\left( {\sin x - \cos x} \right)}^2}}}{2}dx} = \frac{\pi }{2}.\]

由抽屉原理知题给两式不能同时成立.由取等条件知当且仅当

\[f\left( x \right) - \sin x = \cos x - f\left( x \right) \Rightarrow f\left( x \right) = \frac{{\sin x + \cos x}}{2}\]

时取等成立.

5.设$f(x)$在$[0,\infty]$上有$n+1$阶连续导函数,且$f(0)\geq 0,f'(0)\geq0,\ldots,f^{(n)}(0)\geq0.$又对任意$x>0$,有$f(x)\leq f^{(n+1)}(x)$.求证:$f(x)\geq0$.

证.1. (gg)若$f(x)$在$x=0$的某个领域$(0,\xi)$内,满足$f(x)>0$.不妨设存在某个$x>0$,有$f(x)<0$,则此时由连续性,存在某个$x_1>0$,使得$f(x_1)=0$.当$x\in(0,x_1)$时,有$f(x)>0$,则$f^{n+1}(x)\geq f(x)>0$.易推得$f(x)$在$(0,x_1)$上为增函数,$f(x_1)>0$,故此时假设不成立;

2. 若$f(x)$在$x=0$的某个邻域$(0,\xi)$内,满足$f(x)<0$,下证矛盾.构造$g\left( x \right) = \sum\limits_{k = 0}^n {{f^{\left( k \right)}}\left( x \right)} /{e^x}$,则$g(x)$为增函数,所以$g\left( x \right) \ge g\left( 0 \right) \ge 0 \Rightarrow \sum\limits_{k = 0}^n {{f^{\left( k \right)}}\left( x \right)} \ge 0$.由于$f(x)$在$x=0$的某个邻域$(0,\xi)$内,满足$f(x)<0$,则必存在某个$\xi_1$,使得当$x\in(0,\xi_1)$时,有$f(x)<0$.对$k=1,2,\ldots,n$,均存在$\xi_k>0$,使得当$x\in(0,\xi_k)$时,使得$f^{(k)}(x)<0$.取$\eta=\min\{\xi_1,\xi_2,\ldots,\xi_n\}$.当$x\in(0,\eta)$时,有$\sum\limits_{k = 0}^n {{f^{\left( k \right)}}\left( x \right)}<0$,矛盾;

3. 若$f(x)$在$x=0$的某个邻域$(0,\xi)$内,满足$f(x)=0$,且$f(x)$不恒为0,易知此时可推得$(0,\xi)$内,有$f^{(k)}(x)=0,k=1,2,3,\ldots,n$.可转化为在$x=\xi$为初始点的情况,这时我们可采用类似1.,2.的讨论;

4. 若$f(x)=0$,则命题得证.

综上,命题成立.

------------------------------------------

(逆蝶)对于$x\in(0,1]$,由Taylor公式,存在$x_1\in(0,1)$使\[f\left( x \right) = f\left( 0 \right) + f'\left( 0 \right)x + \cdots + \frac{{{f^{\left( n \right)}}\left( 0 \right)}}{{n!}}{x^n} + \frac{{{f^{\left( {n + 1} \right)}}\left( {{x_1}} \right)}}{{\left( {n + 1} \right)!}}{x^{n + 1}}.\]根据条件得\[f\left( x \right) \ge f\left( {{x_1}} \right)\frac{{{x^{n + 1}}}}{{\left( {n + 1} \right)!}}.\]同样将$f(x_1)$展开,可得$x_2\in(0,x_1)$使得\[f\left( {{x_1}} \right) \ge f\left( {{x_2}} \right)\frac{{x_1^{n + 1}}}{{\left( {n + 1} \right)!}}.\]继续这个过程,可得$(0,x)$中严格递减序列$\{x_k\}$使得\[f\left( {{x_k}} \right) \ge f\left( {{x_{k + 1}}} \right)\frac{{x_k^{n + 1}}}{{\left( {n + 1} \right)!}}.\]于是\[f\left( x \right) \ge f\left( {{x_{k + 1}}} \right)\frac{{{x^{n + 1}}}}{{\left( {n + 1} \right)!}}\frac{{x_1^{n + 1}}}{{\left( {n + 1} \right)!}} \cdots \frac{{x_k^{n + 1}}}{{\left( {n + 1} \right)!}}.\]因为$x$及$x_k$都在$[0,1]$中,上式右端当$k\to+\infty$时趋于0,于是对于$x\in[0,1]$有$f(x)\geq0$.由此\[f'\left( x \right) = f'\left( 0 \right) + f''\left( 0 \right)x + \cdots + \frac{{{f^{\left( n \right)}}\left( 0 \right)}}{{\left( {n - 1} \right)!}}{x^{n - 1}} + \frac{{{f^{\left( {n + 1} \right)}}\left( \xi \right)}}{{n!}}{x^n} \ge \frac{{f\left( \xi \right)}}{{n!}}{x^n} \ge 0,\]其中$\xi\in(0,x)$.归纳可证$f^{(k)}\geq0,x\in[0,1],k=1,2,\ldots,n+1$.对函数$g(x)=f(x+1)$重复以上过程可知$f(x)\geq0,x\in[1,2]$.用归纳法可证对任意自然数$m$,$f(x)$在$[m,m+1]$上非负.于是结论得证.

6. 设$f(x)$是$[0,+\infty)$上连续函数,满足$0<f(x)<1$,而且无穷积分在$\int_0^{+\infty}{f(x)\, dx}$和$\int_0^{+\infty}{xf(x)\, dx}$都收敛.求证:\[\int_0^{ + \infty } {xf\left( x \right)\, dx} > \frac{1}{2}{\left( {\int_0^{ + \infty } {f\left( x \right)\, dx} } \right)^2}.\]

证.令\[g\left( y \right) = \int_0^y {xf\left( x \right)dx} - \frac{1}{2}{\left( {\int_0^y {f\left( x \right)dx} } \right)^2},y>0.\]

我们得到\[g'\left( y \right) = yf\left( y \right) - f\left( y \right)\int_0^y {f\left( x \right)dx} = f\left( y \right)\left( {y - \int_0^y {f\left( x \right)dx} } \right).\]

又$0<f(x)<1$,我们得到\[0 < \int_0^y {f\left( x \right)dx} < \int_0^y {dx} = y.\]因此我们有$g'(y)>0,g(y)>g(0)=0.$再令$y\to+\infty$即可.

7. 设$0<\alpha\leq1,\beta>0,\alpha+\beta>1$,$f(x)$是$[1,+\infty)$的正函数,且$\int_1^{+\infty}{f(x)\, dx}$收敛.求证:$\int_1^{+\infty}{\frac{{(f(x))}^\alpha}{x^\beta}\, dx}$收敛.

证.当$\alpha=1$时,由\[0 < \int_1^{ + \infty } {\frac{{f\left( x \right)}}{{{x^\beta }}}dx} \le \int_1^{ + \infty } {f\left( x \right)dx} .\]可知积分收敛.

当$0<\alpha<1$时.

(Holder积分不等式)若函数$f(x)$与$g(x)$在区间$[a,b]$上连续非负,且$p>1,\frac1p+\frac1q=1$,则有不等式\[\int_a^b {f\left( x \right)g\left( x \right)dx} \le {\left( {\int_a^b {{{\left[ {f\left( x \right)} \right]}^p}dx} } \right)^{\frac{1}{p}}}{\left( {\int_a^b {{{\left[ {g\left( x \right)} \right]}^q}dx} } \right)^{\frac{1}{q}}}.\]

取$p=\frac1\alpha,q=\frac1{1-\alpha},f(x)={[f(x)]}^\alpha,g(x)=\left(\frac1x\right)^\beta$,对于任给的正数$A$,我们有

\[0 < \int_1^A {\frac{{{{\left( {f\left( x \right)} \right)}^\alpha }}}{{{x^\beta }}}dx} \le {\left( {\int_1^A {f\left( x \right)dx} } \right)^\alpha }{\left( {\int_1^A {\frac{1}{{{x^{\frac{\beta }{{1 - \alpha }}}}}}dx} } \right)^{1 - \alpha }} \le {\left( {\int_1^{ + \infty } {f\left( x \right)dx} } \right)^\alpha }{\left( {\int_1^{ + \infty } {\frac{1}{{{x^{\frac{\beta }{{1 - \alpha }}}}}}dx} } \right)^{1 - \alpha }}.\]

再注意到$\int_1^{ + \infty } {f\left( x \right)dx} ,\int_1^{ + \infty } {\frac{1}{{{x^{\frac{\beta }{{1 - \alpha }}}}}}dx} \left( {\frac{\beta }{{1 - \alpha }} > 1} \right)$均收敛即可得证.

8. 设$\{a_n\}$是正的递增数列.求证:级数$\sum\limits_{n=1}^\infty \left(\frac{a_{n+1}}{a_n}-1\right)$收敛的充分必要条件是$\{a_n\}$有界.

证.1. ($\Rightarrow$)注意到\[\ln(1+x)<x,x>0.\]我们有\[A = \sum\limits_{n = 1}^\infty {\left( {\frac{{{a_{n + 1}}}}{{{a_n}}} - 1} \right)} > \sum\limits_{n = 1}^\infty {\ln \frac{{{a_{n + 1}}}}{{{a_n}}}} = \mathop {\lim }\limits_{n \to \infty } \ln \frac{{{a_n}}}{{{a_1}}}.\]故\[\mathop {\lim }\limits_{n \to \infty } {a_n} = B \le {a_1}{e^A}.\]

2. ($\Leftarrow$)又\[\sum\limits_{n = 1}^\infty {\left( {\frac{{{a_{n + 1}}}}{{{a_n}}} - 1} \right)} = \sum\limits_{n = 1}^\infty {\frac{{{a_{n + 1}} - {a_n}}}{{{a_n}}}} < \sum\limits_{n = 1}^\infty {\frac{{{a_{n + 1}} - {a_n}}}{{{a_1}}}} = \mathop {\lim }\limits_{n \to \infty } \frac{{{a_n} - {a_1}}}{{{a_1}}} = \frac{1}{{{a_1}}}\mathop {\lim }\limits_{n \to \infty } {a_n} - 1.\]故$\sum\limits_{n=1}^\infty \left(\frac{a_{n+1}}{a_n}-1\right)$收敛.

9. 设$\alpha>0,\{a_n\}$是递增正数列.求证:级数$\sum\limits_{n=1}^\infty \frac{a_{n+1}-a_n}{a_{n+1}a_n^\alpha}$收敛.

证.1. 当$0<\alpha<1$时,有\[\sum\limits_{n = 1}^\infty {\frac{{{a_{n + 1}} - {a_n}}}{{{a_{n + 1}}a_n^\alpha }}} \le \sum\limits_{n = 1}^\infty {\frac{1}{\alpha }\left( {\frac{1}{{a_n^\alpha }} - \frac{1}{{a_{n + 1}^\alpha }}} \right)} = \frac{1}{{\alpha a_1^\alpha }} - \frac{1}{\alpha }\mathop {\lim }\limits_{n \to \infty } \frac{1}{{a_n^\alpha }}.\]

事实上,由Lagrange中值定理,我们有\[\frac{{a_{n + 1}^\alpha - a_n^\alpha }}{{{a_{n + 1}} - {a_n}}} = \alpha {\xi ^{\alpha - 1}} > \alpha a_{n + 1}^{\alpha - 1},\]其中$\xi\in(a_n,a_{n+1})$.

因此,根据$a_n$单调递增的性质,对于其有界和无界两种情况,不等式右端的级数都是收敛的,由此得证.

2. 当$\alpha\geq1$时,又有

\begin{align*}\sum\limits_{n = 1}^\infty {\frac{{{a_{n + 1}} - {a_n}}}{{{a_{n + 1}}a_n^\alpha }}} &= \sum\limits_{n = 1}^\infty {\left( {\frac{1}{{a_n^\alpha }} - \frac{{a_n^{1 - \alpha }}}{{{a_{n + 1}}}}} \right)} \le \sum\limits_{n = 1}^\infty {\left( {\frac{1}{{a_n^\alpha }} - \frac{{a_{n + 1}^{1 - \alpha }}}{{{a_{n + 1}}}}} \right)} \\&= \sum\limits_{n = 1}^\infty {\left( {\frac{1}{{a_n^\alpha }} - \frac{1}{{a_{n + 1}^\alpha }}} \right)} = \frac{1}{{a_1^\alpha }} - \mathop {\lim }\limits_{n \to \infty } \frac{1}{{a_n^\alpha }}.\end{align*}

同理,不等式右端的级数亦是收敛的,证毕.

10. 设$0<\alpha<1$,证明数列\[{a_n} = \frac{1}{{1 + {n^\alpha }}} + \frac{1}{{2 + {n^\alpha }}} + \cdots + \frac{1}{{n + {n^\alpha }}},n = 1,2, \cdots \]发散.

证.注意到\[x>\ln(x+1).\]我们有\[\frac{1}{{k + {n^\alpha }}} > \ln \frac{{k + 1 + {n^\alpha }}}{{k + {n^\alpha }}}.\]

因此\[\mathop {\lim }\limits_{n \to \infty } \sum\limits_{k = 1}^n {\frac{1}{{k + {n^\alpha }}}} > \mathop {\lim }\limits_{n \to \infty } \ln \frac{{n + 1 + {n^\alpha }}}{{1 + {n^\alpha }}} \to \infty .\]

所以此数列发散.

11. 计算\[\mathop {\lim }\limits_{n \to \infty } \left( {\frac{n}{{{1^2} + \sqrt n + {n^2}}} + \frac{n}{{{2^2} + 2\sqrt n + {n^2}}} + \cdots + \frac{n}{{{n^2} + n\sqrt n + {n^2}}}} \right).\]

解.注意到

\begin{align*}&\mathop {\lim }\limits_{n \to \infty } \left[ {\sum\limits_{k = 1}^n {\frac{n}{{{k^2} + {n^2}}}} - \sum\limits_{k = 1}^n {\frac{n}{{{k^2} + k\sqrt n + {n^2}}}} } \right] = \mathop {\lim }\limits_{n \to \infty } \sum\limits_{k = 1}^n {\frac{{kn\sqrt n }}{{\left( {{k^2} + k\sqrt n + {n^2}} \right)\left( {{k^2} + {n^2}} \right)}}} \\&\le \mathop {\lim }\limits_{n \to \infty } \sum\limits_{k = 1}^n {\frac{{kn\sqrt n }}{{{n^2} \cdot {n^2}}}} = \mathop {\lim }\limits_{n \to \infty } \frac{{n\sqrt n }}{{{n^2} \cdot {n^2}}} \cdot \frac{{n\left( {n + 1} \right)}}{2} = 0.\end{align*}

因此

\[\mathop {\lim }\limits_{n \to \infty } \sum\limits_{k = 1}^n {\frac{n}{{{k^2} + k\sqrt n + {n^2}}}} = \mathop {\lim }\limits_{n \to \infty } \sum\limits_{k = 1}^n {\frac{n}{{{k^2} + {n^2}}}} = \mathop {\lim }\limits_{n \to \infty } \frac{1}{n}\sum\limits_{k = 1}^n {\frac{1}{{{{\left( {\frac{k}{n}} \right)}^2} + 1}}} = \int_0^1 {\frac{1}{{{x^2} + 1}}dx} = \frac{\pi }{4}.\]

12. 设$f(x)$是$[0,2\pi]$上可导的凸函数,$f'(x)$有界.试证\[{a_n} = \frac{1}{\pi }\int_0^{2\pi } {f\left( x \right)\cos nx\, dx} \ge 0.\]

证.由分部积分,我们有

\[{a_n} = \frac{1}{\pi }\int_0^{2\pi } {f\left( x \right)\cos nxdx} = \frac{1}{\pi }\int_0^{2\pi } {f\left( x \right)d\frac{{\sin nx}}{n}} = - \frac{1}{{n\pi }}\int_0^{2\pi } {f'\left( x \right)\sin nxdx} .\]

又由第二积分中值定理,我们有

\begin{align*}\int_0^{2\pi } {f'\left( x \right)\sin nxdx} &= f'\left( {0 + } \right)\int_0^\xi {\sin nxdx} + f'\left( {2\pi - } \right)\int_\xi ^{2\pi } {\sin nxdx} \\&= \frac{{1 - \cos n\xi }}{n}\left[ {f'\left( {0 + } \right) - f'\left( {2\pi - } \right)} \right] \le 0.\end{align*}

故\[{a_n} = - \frac{1}{{n\pi }}\int_0^{2\pi } {f'\left( x \right)\sin nxdx} \ge 0.\]

13. 设$\{a_n\}$是正数列使得$\sum\limits_{n=1}^\infty{\frac{1}{a_n}}$收敛.求证\[\sum\limits_{n = 1}^\infty {\frac{n}{{{a_1} + {a_2} + \cdots + {a_n}}}} \le 2\sum\limits_{n = 1}^\infty {\frac{1}{{{a_n}}}} ,\]而且上式右端的系数2是最佳的.

证.由柯西不等式我们得

\[\sum\limits_{m = 1}^n {\frac{{{m^2}}}{{{a_m}}}} \sum\limits_{m = 1}^n {{a_m}}  \ge {\left( {1 + 2 +  \cdots  + n} \right)^2} = \frac{1}{4}{n^2}{\left( {n + 1} \right)^2},\]

即\[\frac{n}{{{a_1} + {a_2} +  \cdots  + {a_n}}} \le \frac{4}{{n{{\left( {n + 1} \right)}^2}}}\sum\limits_{m = 1}^n {\frac{{{m^2}}}{{{a_m}}}} .\]

因此

\begin{align*}\sum\limits_{n = 1}^\infty  {\frac{n}{{{a_1} + {a_2} +  \cdots  + {a_n}}}}  &\le 4\sum\limits_{n = 1}^\infty  {\frac{1}{{n{{\left( {n + 1} \right)}^2}}}\sum\limits_{m = 1}^n {\frac{{{m^2}}}{{{a_m}}}} }  = 4\sum\limits_{m = 1}^\infty  {\frac{{{m^2}}}{{{a_m}}}\sum\limits_{n = m}^\infty  {\frac{1}{{n{{\left( {n + 1} \right)}^2}}}} } \\&\le 4\sum\limits_{m = 1}^\infty  {\frac{{{m^2}}}{{{a_m}}}\sum\limits_{n = m}^\infty  {\frac{1}{2}\left[ {\frac{1}{{{n^2}}} - \frac{1}{{{{\left( {n + 1} \right)}^2}}}} \right]} }  = 2\sum\limits_{m = 1}^\infty  {\frac{1}{{{a_m}}}} .\end{align*}

这里用到了\[\frac{1}{{n{{\left( {n + 1} \right)}^2}}} \le \frac{1}{2}\frac{{2n + 1}}{{{n^2}{{\left( {n + 1} \right)}^2}}} = \frac{1}{2}\left[ {\frac{1}{{{n^2}}} - \frac{1}{{n{{\left( {n + 1} \right)}^2}}}} \right].\]

 

注意到$a_n=n^\alpha,\alpha>1$时有

\[\mathop {\lim }\limits_{\alpha \to 1} \frac{{\sum\limits_{n = 1}^\infty {\frac{n}{{{a_1} + {a_2} + \cdots + {a_n}}}} }}{{\sum\limits_{j = 1}^\infty {\frac{1}{{{a_j}}}} }} = \mathop {\lim }\limits_{\alpha \to 1} \mathop {\lim }\limits_{N \to \infty } \frac{{\sum\limits_{n = 1}^N {\frac{n}{{{1^\alpha } + {2^\alpha } + \cdots + {n^\alpha }}}} }}{{\sum\limits_{n = 1}^N {\frac{1}{{{n^\alpha }}}} }} = \mathop {\lim }\limits_{\alpha \to 1} \mathop {\lim }\limits_{N \to \infty } \frac{{\frac{N}{{\frac{1}{{\alpha + 1}}{N^{\alpha + 1}} + O\left( {{N^\alpha }} \right)}}}}{{\frac{1}{{{N^\alpha }}}}} = 2.\]

14. 设$f(x)$是$[0,+\infty)$上正的连续函数,且$\int_0^{+\infty}{\frac{1}{f(x)}\, dx}$收敛.记$F(x)=\int_0^x{f(t)\, dt}$.求证\[\int_0^{ + \infty } {\frac{x}{{F\left( x \right)}}dx} < 2\int_0^{ + \infty } {\frac{1}{{f\left( x \right)}}dx} ,\]且上式右端的系数2是最佳的.

证.(陈洪葛)由$Cauchy-Schwarz$不等式,得到\[\left( {\int_0^x {f\left( t \right)dt} } \right) \cdot \left( {\int_0^x {\frac{{{t^2}}}{{f\left( t \right)}}dt} } \right) \ge {\left( {\int_0^x {tdt} } \right)^2} = \frac{1}{4}{x^4}.\]

所以

\[\int_0^{ + \infty } {\frac{x}{{F\left( x \right)}}dx} \le \int_0^{ + \infty } {\frac{4}{{{x^3}}}\int_0^x {\frac{{{t^2}}}{{f\left( t \right)}}dt} dx} .\]

注意到\[\mathop {\lim }\limits_{x \to {0^ + }} \frac{1}{{{x^2}}}\int_0^x {\frac{{{t^2}}}{{f\left( t \right)}}dt} = \mathop {\lim }\limits_{x \to 0} \frac{x}{{2f\left( x \right)}} = 0.\]

以及\[\frac{1}{{{x^2}}}\int_0^x {\frac{{{t^2}}}{{f\left( t \right)}}dt} = \int_0^\xi {\frac{1}{{f\left( t \right)}}dt} < \int_0^{ + \infty } {\frac{1}{{f\left( t \right)}}dt} .\]

故\begin{align*}&\int_0^A {\frac{4}{{{x^3}}}\int_0^x {\frac{{{t^2}}}{{f\left( t \right)}}dt} dx} = \int_0^A {\left( {\int_0^x {\frac{{{t^2}}}{{f\left( t \right)}}dt} } \right)d\left( { - \frac{2}{{{x^2}}}} \right)} \\&= \int_0^A {\frac{2}{{f\left( x \right)}}dx} - \frac{2}{{{A^2}}}\int_0^A {\frac{{{t^2}}}{{f\left( t \right)}}dt} < \int_0^A {\frac{2}{{f\left( x \right)}}dx} - 2\int_\xi ^A {\frac{{{t^2}}}{{f\left( t \right)}}dt}.\end{align*}

令$A\to+\infty$,得到

\[\int_0^{ + \infty } {\frac{4}{{{x^3}}}\int_0^x {\frac{{{t^2}}}{{f\left( t \right)}}dt} dx} \le \int_0^{ + \infty } {\frac{2}{{f\left( x \right)}}dx} - 2\int_\xi ^{ + \infty } {\frac{{{t^2}}}{{f\left( t \right)}}dt} < \int_0^{ + \infty } {\frac{2}{{f\left( x \right)}}dx}.\]

另外,当我们取$f(x)=x^a+1(a>1)$时,有$\int_0^{ + \infty } {\frac{{dx}}{{{x^a} + 1}}} = \frac{\pi }{{a\sin \frac{\pi }{a}}}$收敛.此时有

\[\mathop {\lim }\limits_{a \to 1} \frac{{\int_0^{ + \infty } {\frac{x}{{F\left( x \right)}}dx} }}{{\int_0^{ + \infty } {\frac{1}{{f\left( x \right)}}dx} }} = \mathop {\lim }\limits_{a \to 1} \frac{{\int_0^{ + \infty } {\frac{x}{{{x^{a + 1}}/\left( {a + 1} \right) + x}}dx} }}{{\int_0^{ + \infty } {\frac{{dx}}{{{x^a} + 1}}} }} = \mathop {\lim }\limits_{a \to 1} \frac{{\int_0^{ + \infty } {\frac{1}{{{x^a}/\left( {a + 1} \right) + 1}}dx} }}{{\int_0^{ + \infty } {\frac{{dx}}{{{x^a} + 1}}} }} = \mathop {\lim }\limits_{a \to 1} {\left( {a + 1} \right)^{\frac{1}{a}}} = 2.\]

15. 设$f(x)$在$\mathbb{R}$上有二阶导函数,$f(x),f'(x),f''(x)$都大于零,假设存在正数$a,b$使得$f''(x)\leq af(x)+bf'(x)$对一切$x\in\mathbb{R}$成立.

1. 求证:$\mathop {\lim }\limits_{x \to - \infty } f'\left( x \right) = 0$;

2. 求证:存在常数$c$使得$f'(x)\leq cf(x)$;

3. 求使上面不等式成立的最小常数$c$.

证.1. 显然由单调有界定理知$\mathop {\lim }\limits_{x \to - \infty } f\left( x \right),\mathop {\lim }\limits_{x \to - \infty } f'\left( x \right)$均存在,不妨设$\mathop {\lim }\limits_{x \to - \infty } f\left( x \right)=c_1,\mathop {\lim }\limits_{x \to - \infty } f'\left( x \right)=c_2$.因此我们有$f(x)\geq c_1\geq0,f'(x)\geq c_2\geq0$.又\[f\left( 0 \right) = \int_x^0 {f'\left( x \right)dx} + f\left( x \right) \ge - {c_2}x + f\left( x \right) \Rightarrow f\left( x \right) \le {c_2}x + f\left( 0 \right).\]我们知$c_2=0$,否则令$x\to-\infty$,矛盾.

2. 令

\begin{align*}&g\left( x \right) = \left( {\frac{{b + \sqrt {4a + {b^2}} }}{2}f\left( x \right) - f'\left( x \right)} \right){e^{\frac{{ - b + \sqrt {4a + {b^2}} }}{2}x}},\\&g'\left( x \right) = \left( {af\left( x \right) + bf'\left( x \right) - f''\left( x \right)} \right){e^{\frac{{ - b + \sqrt {4a + {b^2}} }}{2}x}} \ge 0.\end{align*}

又$\lim\limits_{n\to-\infty} g(x)=0$.故$g(x)\geq g(-\infty)=0$.即\[f'\left( x \right) \le \frac{{b + \sqrt {4a + {b^2}} }}{2}f\left( x \right).\]故存在常数$c={\frac{{b + \sqrt {4a + {b^2}} }}{2}}$使得$f'(x)\leq cf(x)$成立.

 

3. 我觉得应该把“求使上面不等式成立的最小常数$c$”改成“求使上面不等式成立的最大常数$c$”.事实上,我们取:

\[\left\{ \begin{array}{l}h\left( x \right) = {e^{cx}}\\h'\left( x \right) = c{e^{cx}}\\h''\left( x \right) = {c^2}{e^{cx}}\end{array} \right.\left( {c > 0} \right).\]

只需保证\[c^2\leq bc+a\Rightarrow 0<c\leq {\frac{{b + \sqrt {4a + {b^2}} }}{2}}.\]

故常数$c$的最大值为${\frac{{b + \sqrt {4a + {b^2}} }}{2}}$.

16. 设$f(x)$是$\mathbb{R}$上有下界或者有上界的连续函数且存在正数$a$使得\[f\left( x \right) + a\int_{x - 1}^x {f\left( t \right)dt} \]为常数.求证:$f(x)$必为常数.

证.(Slade)由题意,$f(x)$必存在上下界,否则,在等式\[f\left( x \right) + a\int_{x - 1}^x {f\left( t \right)dt}=C\]两端同时取$x\to \infty$,等式左端无界,而右端为常数,矛盾.

再之,注意到$\int_{x-1}^x f(t)\,dt$是可微的,故$f$也是可微的.又\[f'\left( x \right) = a\left( {f\left( {x - 1} \right) - f\left( x \right)} \right) \Rightarrow \left| {f'\left( x \right)} \right| \le a\left( {\left| {f\left( {x - 1} \right)} \right| + \left| {f\left( x \right)} \right|} \right).\]由于$f$有界,故$f'$亦有界.记$N=[a]+1$,则$0<\frac aN<1$.令$x=\frac x N$,我们得到\[f'\left( {\frac{x}{N}} \right) = a\left( {f\left( {\frac{x}{N} - 1} \right) - f\left( {\frac{x}{N}} \right)} \right).\]

固定$x$,由Lagrange中值定理,存在${a_1} \in \left( {\frac{x}{N} - 1,\frac{x}{N}} \right)$使得\[f\left( {\frac{x}{N} - 1} \right) - f\left( {\frac{x}{N}} \right) = - \frac{1}{N}f'\left( {{a_1}} \right),\]即$f'\left( {\frac{x}{N}} \right) = - \frac{a}{N}f'\left( {{a_1}} \right)$.类似地,我们有数列$\{a_n\}$使得\[f'\left( {\frac{x}{N}} \right) = - \frac{a}{N}f'\left( {{a_1}} \right) = {\left( { - \frac{a}{N}} \right)^2}f'\left( {{a_2}} \right) = \cdots = {\left( { - \frac{a}{N}} \right)^n}f'\left( {{a_n}} \right).\]令$n\to\infty$,我们有$f'\left( {\frac{x}{N}} \right) = 0 $即$f'\left( x \right) = 0$.故$f(x)$必为常数.

17. 设$f:[0,+\infty)\to [0,+\infty)$且对任意$x\geq0$有$f\circ f(x)=af(x)+bx$,其中$a<0,b>0$.求$f(x).$

证.先证明几个引理:

引理1.设$f\in C^0(\mathbb{R},\mathbb{R})$是方程$f(f(x))=af(x)+bx$的解.若方程中的常数$b\neq0$,则$f:\mathbb{R}\to\mathbb{R}$即单射又是满射,即是一个一一映射.

引理2.设$f\in C^0(\mathbb{R},\mathbb{R})$是方程$f(f(x))=af(x)+bx$的解.若$b\neq0$,且$\lambda$的多项式$\lambda^2-a\lambda-b$的两个根$r$与$s$不相等,则对任意$x\in\mathbb{R}$及任意$n\in\mathbb{Z}$均有

\[{f^n}\left( x \right) = \frac{{{s^n}\left( {f\left( x \right) - rx} \right) + {r^n}\left( {sx - f\left( x \right)} \right)}}{{s - r}}.\]

事实上,当$n$为非正整数时下一式成立:\[{f^n}\left( x \right) = \frac{{{s^n}\left( {{f^{ - 1}}\left( x \right) - \frac{x}{r}} \right) + {r^n}\left( {\frac{x}{s} - {f^{ - 1}}\left( x \right)} \right)}}{{\frac{1}{s} - \frac{1}{r}}}.\]以$f(x)$代替$x$,以$n-1$代替$n$,可推知$n$为非正整数时亦成立.

引理3.设$g_n\in C^0(\mathbb{R},\mathbb{R})$,且${(-1)}^n g_n(x)$对$x$单调递增($n=0,1,2,\ldots$).若极限$g(x)=\lim\limits_{n\to\infty}g_n(x)$对任$x\in \mathbb{R}$均存在,则极限函数$g$是个常值函数.

事实上,因$g(x)=\lim\limits_{n\to\infty}g_{2n}(x)$,而$g_{2n}(x)$对$x$递增.又因$g(x)=\lim\limits_{n\to\infty}g_{2n+1}(x)$,而$\lim\limits_{n\to\infty}g_{2n+1}(x)$对$x$递减,故$g(x)$对$x$递减.于是$g$只能是个常值函数.

回到原题,我们有定理:设$r$及$s$是$\lambda$的二次多项式$\lambda^2-a\lambda-b=0$的两个根.若$r<0<s$,且$r\neq -s$,则$f\in C^0(\mathbb{R},\mathbb{R})$是方程$f(f(x))=af(x)+bx$的解的充分必要条件是$f(x)=rx$(对任$x\in\mathbb{R}$),或者$f(x)=sx$(对任$x\in\mathbb{R}$),或者,当$s=1$时还可以是$f(x)=rx+c$(对任$x\in\mathbb{R}$,$c$可以是任一个给定的实数).

定理的充分性部分是显然的.下面证明定理的必要性部分,设$f$是方程$f(f(x))=af(x)+bx$的一个解,据引理1知$f$严格单调.若$f$严格递增,可推出

\[sx - f\left( x \right) = \left\{ \begin{array}{l}\mathop {\lim }\limits_{n \to \infty } \frac{{\left( {s - r} \right){f^n}\left( x \right)}}{{{r^n}}},\text{若}\left| r \right| > s,\\\mathop {\lim }\limits_{n \to - \infty } \frac{{\left( {s - r} \right){f^n}\left( x \right)}}{{{r^n}}},\text{若}\left| r \right| < s.\end{array} \right.\]

因${(-1)}^n(s-r)f^n(x)/r^n$是$x$的递增函数(任意$n\in\mathbb{Z}$),由引理3可知极限函数$sx-f(x)$恒取常值,即存在某$c\in\mathbb{R}$使得$f(x)=sx+c$对任意$x\in\mathbb{R}$成立,把$f(x)$的这一表达式代入得到$s^2x+sc+c=(r+s)(sx+c)-rsx$,推出$c=0$.因此,当$f$递增时$f(x)=sx$(对任意$x\in\mathbb{R}$).

若$f$严格递减,可推出\[f\left( x \right) - rx = \left\{ \begin{array}{l}\mathop {\lim }\limits_{n \to \infty } \frac{{\left( {s - r} \right){f^n}\left( x \right)}}{{{s^n}}},s > \left| r \right|,\\\mathop {\lim }\limits_{n \to - \infty } \frac{{\left( {s - r} \right){f^n}\left( x \right)}}{{{s^n}}},s < \left| r \right|.\end{array} \right.\]

因${(-1)}^n(s-r)f^n(x)/s^n$是$x$的递增函数(任意$n\in\mathbb{Z}$),由引理3可同样推出$f(x)=rx+c$(对任意$x\in\mathbb{R}$).将此表达式代入得到$r^2x+rc+c=(r+s)(rx+c)-rsx$,可化简为$(s-1)c=0$.因此,当$s\neq1$时$c=0$,当$s=1$时$c$可为任意给定的实数.

另外:注意到$f:[0,+\infty)\to [0,+\infty)$这一条件,我们知$f(x)=sx(s>0)$.可参阅:

[1]关于迭代函数方程$f^2(x)=af(x)+bx$的通解,麦结华,数学研究与评论第17卷第1期83-90页,1997年2月.

[2]J.Matkowski and Zhang Weinian, Method of characteristics for functional equations in polynomial

form, Acta Math.Sinica, New Series.

18. 设$f(x)$在$(-\infty,+\infty)$上连续,且对任意$x$有$f(2x-f(x))=x$.求证:$f(x)\equiv x+c$,其中$c$为常数.

证.(逆蝶)令$g(x)=2x-f(x)$,则$f[g(x)]=x$,显然$g$是单射且为增函数.若$g$有上界,则$\lim\limits_{x\to+\infty}{g(x)}=a$存在.又$f$连续,$\lim\limits_{x\to+\infty}{f[g(x)]}=f(a)$,矛盾.从而$g$无上界也无下界.由于$g$连续,$\forall x\in R,\exists y$使得$x=g(y)$.从而$f(x)=f[g(y)]=y$,于是$g[f(x)]=g(y)=x$.由$g$递增知$f$递增.

若$\exists x_1,x_2$使得$f(x_1)-x_1\neq f(x_2)-x_2$,则由于$f(x)-x$的连续性知其可以取遍$f(x_1)-x_1$及$f(x_2)-x_2$之间的任何数.

设$d,d'$在它们之间,且$d/d'>1$是正无理数,设$d=f(x)-x,d'=f(x')-x'$.令$x_0=x,x_{n+1}=2x_n-f(x_n)$,则$f(x_{n+1})=x_n$.归纳可得$x_n=x+nd$,同理$x'_n=x'+nd'$.

令$x_m>x'_n,x_{m+1}<x'_n+1$,即$x+md>x'+nd,x+d+md<x'+d'+nd'$.即$(x'-x)/d<(md')/d-n<(x'-x)/d+(d/d'-1)$.由$d/d'$为无理数知上式关于$m,n$有解,这与$f$递增矛盾.

下面是一个推广:对$\forall m\neq0$,若一个连续函数$f:\mathbb{R}\to\mathbb{R}$满足函数方程\[f\left(2x-\frac{f(x)}{m}\right)=mx,\]则有$f(x)=m(x-c)$.

证.(陈洪葛)我们设$g(x)=2x-\frac{f(x)}{m}$,显然$g(x)$是连续函数且有\[g(g(x))=2g(x)-x,\forall x\in\mathbb{R}.\]

若$g(x_1)=g(x_2)$,则有$g(g(x_1))=g(g(x_2))$,我们得到\[x_1=x_2.\]故$g(x)$是一个单射,而我们知道,若$g(x)$是一个连续的单射,则$g(x)$严格单调(关于这点可以用反证法证明).因此,$g(x)$有2种情况,严格递增或者严格递减.下面证明$g(x)$只能严格递增.

(反证)设$g(x)$严格递减,则对于$x_1<x_2$,我们有$g(x_1)>g(x_2)$,接着又有$g(g(x_1))<g(g(x_2))$,二者等价于\[2g(x_1)-x_1<2g(x_2)-x_2\Leftrightarrow 2[g(x_1)-g(x_2)]<x_1-x_2.\]上面不可能成立,因为左边大于0而右边小于0,故$g(x)$只能严格递增.

改写$g(g(x))=2g(x)-x$为\[g(g(x))-g(x)=g(x)-x.\]递推后得到\[g^n(x)=ng(x)-(n-1)x,(n\geq1),\]这里$g^{(n)}(x)$表示$n$次复合.那么有\[g^n(x)-g^n(0)=n[g(x)-x-g(0)]+x\Leftrightarrow\frac{g^n(x)-g^n(0)}{n}=g(x)-x-g(0)+\frac{x}{n}.\]而$g(x)$严格递增,$g^n(x)$也严格递增,故对上式令$n\to\infty$,由$g(x)$的单调性,我们得到

\begin{align*}g(x)&\leq x+g(0),&&x<0\\g(x)&\geq x+g(0),&&x>0.\end{align*}

这样,我们得到$g(x)$的值域也是$\mathbb{R}$,故$g(x)$是一个一一映射,且$g^{-1}$存在.现在,用$x=g^{-1}(g^{-1}(y))$代入原来的方程,则有\[g^{-1}(g^{-1}(y))=2g^{-1}(y)-y.\]$g^{-1}(y)$同样满足这个方程,则用相同的手段,我们得到

\begin{align*}g^{-1}(y)&\leq y+g(0),&&y<0\\g^{-1}(y)&\geq y+g(0),&&y>0.\end{align*}

现在,用$x=g^{-1}(y)$代入\[g(g(x))-g(x)=g(x)-x\]得到\[g(y)-y=y-g^{-1}(y).\]令$y=0$得到$g^{-1}(0)=-g(0)$.

假设$g(0)\geq0$,则对$x>0$有$g(x)\geq x+g(0)>0$,则对$y=g(x)>0$有$x>g(x)+g^{-1}(0)=g(x)-g(0)$.故得到

\[g(x)=x+g(0),x>0\]同理可得\[g(x)=x+g(0),x<0.\]这样我们得到$f(x)=m(x-g(0))$对$x\in\mathbb{R}$成立.

19. 设$0<a<2$.求证:不存在$(-\infty,+\infty)$上连续的函数$f(x)$,使得对任意$x$有$f(ax-f(x))=x$.

证.我们设$g(x)=ax-f(x)$,显然$g(x)$是连续函数且有\[g(g(x))=ag(x)-x,\forall x\in\mathbb{R}.\]假设此方程有一连续函数解$g:\mathbb{R}\to\mathbb{R}$且多项式方程$r^2-ar+1=0$有一对复数特征根\[{r_1} = a - ib = S\exp \left( { - i\theta } \right),{r_2} = a + ib = S\exp \left( {i\theta } \right),\]其中$a,b\in \mathbb{R},b>0,S>0$及$\theta\in(0,\pi)$.易知$f$是单调的且$f^2$是严格递增的.并且对于$x\neq0$有$f(x)\neq x$.因此当$f$是严格递增时,数列$\left\{ {{f^{n + 1}}\left( x \right)-{f^n}\left( x \right)} \right\}$对于任意固定的$x\neq0$同样是严格递增的.因此,我们有

\begin{align*}{f^n}\left( x \right) &= \frac{{r_2^n}}{{{r_2} - {r_1}}}\left( {f\left( x \right) - {r_1}x} \right) + \frac{{r_1^n}}{{{r_2} - {r_1}}}\left( {{r_2}x - f\left( x \right)} \right)\\&= \frac{1}{b}{S^n}\sin \theta \cdot f\left( x \right) - \frac{1}{b}{S^{n + 1}}\sin \left( {n - 1} \right)\theta \cdot x.\end{align*}

\[{f^{n + 1}}\left( x \right) - {f^n}\left( x \right) = r_2^nU\left( x \right) + r_1^nV\left( x \right),\]

其中$U\left( x \right) = \frac{{{r_2} - 1}}{{{r_2} - {r_1}}}\left( {f\left( x \right) - {r_1}x} \right),V\left( x \right) = \frac{{{r_1} - 1}}{{{r_2} - {r_1}}}\left( {{r_2}x - f\left( x \right)} \right)$.显然$\overline U\left( x \right) = V\left( x \right)$,故对于固定的$x\neq0$我们可令

\[U\left( x \right) = T\exp \left( {it} \right)\text{和}V\left( x \right) = T\exp \left( { - it} \right),\]

其中$T\geq0$且$t\in[0,2\pi]$.因此\[\begin{array}{l}{f^{n + 1}}\left( x \right) - {f^n}\left( x \right) = {S^n}T\left[ {\exp \left( {i\left( {n\theta + t} \right)} \right) + \exp \left( { - i\left( {n\theta + t} \right)} \right)} \right]\\= 2{S^n}T\cos \left( {n\theta + t} \right)\end{array}.\]

因为当$T>0$时$S>0$,与数列$\left\{ {{f^{n + 1}}\left( x \right)-{f^n}\left( x \right)} \right\}$的性质矛盾;当$T=0$时我们有$U(x)=V(x)=0$,因此对所有$x\neq0$我们有$f(x)=r_1x=r_2x$,由此我们得到$r_1=r_2$这一矛盾的结论.证毕.

20. 求证:不存在可微函数$f:(0,+\infty)\to(0,+\infty)$满足方程\[f'(x)=f\circ f(x),x\in(0,+\infty).\]

证.(刘畅)由$f(x)>0$知$f(f(x))>0$,从而有$f'(x)>0$,显然$\lim\limits_{x\to+\infty}f(x)=+\infty$(否则得到$\lim\limits_{x\to+\infty}f'(x)=0$,矛盾),因此$\lim\limits_{x\to+\infty}f'(x)=\lim\limits_{x\to+\infty}f(f(x))=+\infty$.当$x$充分大时,我们有$f'(x)>1$,故存在足够大的$n\in\mathbb{N_+}$使得$f(n)>n+1$,我们有\[f\left( {n + 1} \right) - f\left( n \right) = \int_n^{n + 1} {f'\left( x \right)dx} = \int_n^{n + 1} {f\left( {f\left( x \right)} \right)dx} > f\left( {n + 1} \right),\]矛盾.

21. 设正数列$\{a_n\}$满足$\varliminf\limits_{n\to+\infty}{a_n}=1,\varlimsup\limits_{n\to+\infty}{a_n}<+\infty,\lim\limits_{n\to+\infty}\sqrt[n]{a_1a_2\ldots a_n}=1.$求证:

\[\lim\limits_{n\to+\infty}\frac{a_1+a_2+\ldots+a_n}{n}=1.\]

证.(别人解答)记$x_n=\ln a_n$,由题设条件,

\begin{align*}&\varliminf\limits_{n\to+\infty}x_n=0,\varlimsup\limits_{n\to+\infty}x_n\leq A<+\infty(A>0),\\&\lim\limits_{n\to\infty}\frac1n\sum_{k=1}^n x_k=0.\end{align*}

假设所有的$x_n\geq0$,则当$x\leq\ln2$时,成立不等式$e^x\leq1+2x$.对于固定的$n$,记${S_n} = \left\{ {i \in \mathbb{Z}\left| {1 \le i \le n,{x_i} \le \ln 2} \right.} \right\},{T_n} = \left\{ {i \in \mathbb{Z}\left| {1 \le i \le n,{x_i} > \ln 2} \right.} \right\}$,则

\[\frac{1}{n}\sum\limits_{k = 1}^n {{x_k}} = \frac{1}{n}\sum\limits_{k \in {S_n}} {{x_k}} + \frac{1}{n}\sum\limits_{k \in {T_n}} {{x_k}} \ge \frac{{\left| {{T_n}} \right|}}{n}\ln 2 \ge 0.\]

由此即知\[\mathop {\lim }\limits_{n \to \infty } \frac{{\left| {{T_n}} \right|}}{n} = 0.\]

从而\[\frac{1}{n}\sum\limits_{k = 1}^n {{e^{{x_k}}}} = \frac{1}{n}\sum\limits_{k \in {S_n}} {{e^{{x_k}}}} + \frac{1}{n}\sum\limits_{k \in {T_n}} {{e^{{x_k}}}} \le 1 +\frac{{\left| {{T_n}} \right|}}{n}\left( {C+ {e^A}} \right) + \frac{2}{n}\sum\limits_{k = 1}^n {{x_k}} .\]而\[\frac{1}{n}\sum\limits_{k = 1}^n {{e^{{x_k}}}} \ge {e^{\frac{1}{n}\sum\limits_{k = 1}^n {{x_k}} }}.\]由迫敛性即得\[\mathop {\lim }\limits_{n \to \infty } \frac{1}{n}\sum\limits_{k = 1}^n {{e^{{x_k}}}} = \mathop {\lim }\limits_{n \to \infty } \frac{1}{n}\sum\limits_{k = 1}^n {{a_k}} = 1.\]在一般情况下,作序列\[{z_n} = \left\{ \begin{array}{l}- {x_n},{x_n} < 0\\0,{x_n} \ge 0\end{array} \right.,n = 1,2, \cdots \]由题设可知\[\mathop {\lim }\limits_{n \to \infty } {z_n} = 0.\]故$y_n=x_n+z_n\geq0$,得到$\mathop {\lim }\limits_{n \to \infty } \frac{1}{n}\sum\limits_{k = 1}^n {{y_k}} = 0.$从而$\mathop {\lim }\limits_{n \to \infty } \frac{1}{n}\sum\limits_{k = 1}^n {{e^{{y_k}}}} = 1$,再由$\frac{1}{n}\sum\limits_{k = 1}^n {{e^{{x_k}}}} \le \frac{1}{n}\sum\limits_{k = 1}^n {{e^{{y_k}}}}$,可得\[\mathop {\lim }\limits_{n \to \infty } \frac{1}{n}\sum\limits_{k = 1}^n {{e^{{x_k}}}} = \mathop {\lim }\limits_{n \to \infty } \frac{1}{n}\sum\limits_{k = 1}^n {{a_k}} = 1.\]

22. 设$f(x)$在$\mathbb{R}$上有二阶连续导数且满足方程\[f^3+{(f')}^3=1.\]求证:$f=1$.

证.(刘畅)显然$f=1$满足题意.当$f\neq1$时,我们有

\[f' = \sqrt[3]{{1 - {f^3}}},f'' = \frac{{ - {f^2}f'}}{{\sqrt[3]{{{{\left( {1 - {f^3}} \right)}^2}}}}} = \frac{{ - {f^2}}}{{\sqrt[3]{{1 - {f^3}}}}}.\]

若$\exists x_0\in\mathbb{R}$使$f(x_0)>1$,则$f'(x_0)<0$,故对$x\leq x_0$都有$f'(x)<0,f''(x)>0$,从而$f(-\infty)=+\infty$.若$\exists x_1>x_0$使得$f(x_1)<1$,则对$x\leq x_1$都有$f'(x)>0,f''(x)<0$,故$f(-\infty)=-\infty$,矛盾.

故对$x>x_0$,均有$f(x)\geq1$.

若$\exists a>x_0$使得$f(a)=1$(其中$a$为使得$f(x)=1$的最小实数),则我们有$x>a$均有$f(x)=1$,这时有$f''(x)$在$x=a$处不连续,矛盾.

若不$\exists b>x_0$使得$f(b)=1$,则对于$x\in\mathbb{R}$均有$f(x)>1$,设$f(+\infty)=1$(否则有$\mathop {\lim }\limits_{x \to + \infty } f'\left( x \right) < 0$,与$\mathop {\lim }\limits_{x \to + \infty } f'\left( x \right) = 0$矛盾),而\[\mathop {\lim }\limits_{x \to + \infty } f'\left( x \right) = 0,\mathop {\lim }\limits_{x \to + \infty } f''\left( x \right) = + \infty .\]亦矛盾.证毕.


 

 

 

PDF下载:http://pan.baidu.com/s/1jGqVEoY

 

一道与ODE有关的数列极限求解

一边参考葛神的日志,一边把这题做出来了,在今年中秋节这算是给我的不错礼物.

(准大一问题)已知${U_{n + 2}} = {U_{n + 1}} + \frac{{{U_n}}}{{2n}},{U_1} = 0,{U_2} = 1$,求$\mathop {\lim }\limits_{n \to \infty } \frac{{{U_n}}}{{\sqrt n }}.$
解.设$S\left( x \right) = {U_1} + {U_2}x +  \cdots  + {U_n}{x^{n - 1}} +  \cdots $,则
\begin{align*}S\left( x \right) &= {U_1} + {U_2}x + \sum\limits_{n = 1}^\infty  {{U_{n + 2}}{x^{n + 1}}} \\&= {U_1} + {U_2}x + \sum\limits_{n = 1}^\infty  {\left( {{U_{n + 1}} + \frac{{{U_n}}}{{2n}}} \right){x^{n + 1}}} \\&= {U_1} + {U_2}x + x\left( {S\left( x \right) - {U_1}} \right) + \frac{1}{2}x\int_0^x {S\left( t \right)dt} \\&= x\left( {S\left( x \right) + 1} \right) + \frac{1}{2}x\int_0^x {S\left( t \right)dt}.\end{align*}
\[\frac{{S\left( x \right)}}{x} = S\left( x \right) + 1 + \frac{1}{2}\int_0^x {S\left( t \right)dt} .\]
两边同时求导
\[\frac{{xS'\left( x \right) - S\left( x \right)}}{{{x^2}}} = S'\left( x \right) + \frac12 S\left( x \right) \Rightarrow S = \frac{x}{{{{\left( {1 - x} \right)}^{\frac{3}{2}}}}}{e^{ - \frac{x}{2}}}.\]
注意到
\[\frac{1}{{{{\left( {1 - x} \right)}^{\frac{3}{2}}}}} = \sum\limits_{n = 0}^\infty  {\frac{{\left( {2n + 1} \right)!!}}{{{2^n}n!}}{x^n}} ,{e^{ - \frac{x}{2}}} = \sum\limits_{n = 0}^\infty  {{{\left( { - 1} \right)}^n}\frac{1}{{{2^n}n!}}{x^n}} .\]
\[\frac{x}{{{{\left( {1 - x} \right)}^{\frac{3}{2}}}}}{e^{ - \frac{x}{2}}} = \sum\limits_{n = 0}^\infty  {{c_n}{x^n}} ,\]
其中
\[{c_n} = \sum\limits_{k = 0}^{n - 2} {\frac{{{{\left( { - 1} \right)}^k}\left( {2n - 2k - 3} \right)!!}}{{{2^{n - 2}}k!\left( {n - k - 2} \right)!}}} .\]
\[\mathop {\lim }\limits_{n \to \infty } \frac{{{U_n}}}{{\sqrt n }} = \mathop {\lim }\limits_{n \to \infty } \frac{{\sum\limits_{k = 0}^{n - 2} {\frac{{{{\left( { - 1} \right)}^k}\left( {2n - 2k - 3} \right)!!}}{{{2^{n - 2}}k!\left( {n - k - 2} \right)!}}} }}{{\sqrt n }} = \mathop {\lim }\limits_{n \to \infty } \frac{1}{{{2^{n - 2}}\sqrt n }}\frac{{\left( {2n - 3} \right)!!}}{{\left( {n - 2} \right)!}} \cdot \sum\limits_{k = 0}^{n - 2} {\frac{{{{\left( { - 1} \right)}^k}}}{{k!}}} .\]
注意到
\[\mathop {\lim }\limits_{n \to \infty } \frac{1}{{{2^{n - 2}}\sqrt n }}\frac{{\left( {2n - 3} \right)!!}}{{\left( {n - 2} \right)!}} = \frac{2}{{\sqrt \pi  }},\mathop {\lim }\limits_{n \to \infty } \sum\limits_{k = 0}^{n - 2} {\frac{{{{\left( { - 1} \right)}^k}}}{{k!}}}  = \frac{1}{e}.\]
事实上
\begin{align*}&\mathop {\lim }\limits_{n \to \infty } \frac{1}{{{2^{n - 2}}\sqrt n }}\frac{{\left( {2n - 3} \right)!!}}{{\left( {n - 2} \right)!}} = \mathop {\lim }\limits_{n \to \infty } \frac{1}{{{2^{n - 2}}\sqrt n }}\frac{{\left( {2n - 2} \right)!}}{{\left( {n - 2} \right)!\left( {2n - 2} \right)!!}}\\&= \mathop {\lim }\limits_{n \to \infty } \frac{1}{{{2^{n - 2}}\sqrt n }}\frac{{\left( {2n - 2} \right)!}}{{\left( {n - 2} \right)!{2^{n - 1}}\left( {n - 1} \right)!}} = \mathop {\lim }\limits_{n \to \infty } \frac{1}{{{2^{2n - 3}}\sqrt n }}\frac{{\left( {2n - 2} \right)!}}{{\left( {n - 2} \right)!\left( {n - 1} \right)!}}\\&= \mathop {\lim }\limits_{n \to \infty } \frac{1}{{{2^{2n - 3}}\sqrt n }} \cdot \frac{{{{\left( {2n - 2} \right)}^{2n - 2}}\sqrt {2\pi \left( {2n - 2} \right)} }}{{{e^{2n - 2}}}} \cdot \frac{{{e^{n - 2}}}}{{{{\left( {n - 2} \right)}^{n - 2}}\sqrt {2\pi \left( {n - 2} \right)} }} \cdot \frac{{{e^{n - 1}}}}{{{{\left( {n - 1} \right)}^{n - 1}}\sqrt {2\pi \left( {n - 1} \right)} }}\\&= \mathop {\lim }\limits_{n \to \infty } \frac{1}{{{2^{2n - 3}}}} \cdot \frac{{{{\left( {2n - 2} \right)}^{2n - 2}}}}{{{{\left( {n - 2} \right)}^{n - 2}}{{\left( {n - 1} \right)}^{n - 1}}}} \cdot \frac{{\sqrt {2\pi \left( {2n - 2} \right)} }}{{\sqrt {2\pi \left( {n - 2} \right)} \sqrt {2\pi \left( {n - 1} \right)} \sqrt n }} \cdot \frac{1}{e}\\&= \frac{1}{{e\sqrt \pi  }}\mathop {\lim }\limits_{n \to \infty } \frac{{{2^{2n - 2}}}}{{{2^{2n - 3}}}} \cdot \frac{{{{\left( {n - 1} \right)}^{n - 1}}}}{{{{\left( {n - 2} \right)}^{n - 2}}}} \cdot \frac{1}{{\sqrt {n\left( {n - 2} \right)} }} = \frac{2}{{e\sqrt \pi  }}\mathop {\lim }\limits_{n \to \infty } \frac{{{{\left( {n - 1} \right)}^{n - 2}}}}{{{{\left( {n - 2} \right)}^{n - 2}}}} \cdot \frac{{\left( {n - 1} \right)}}{{\sqrt {n\left( {n - 2} \right)} }}\\&= \frac{2}{{e\sqrt \pi  }}\mathop {\lim }\limits_{n \to \infty } {\left( {1 + \frac{1}{{n - 2}}} \right)^{n - 2}} = \frac{2}{{\sqrt \pi  }}.\end{align*}
因此
\[\mathop {\lim }\limits_{n \to \infty } \frac{{{U_n}}}{{\sqrt n }} = \mathop {\lim }\limits_{n \to \infty } \frac{1}{{{2^{n - 2}}\sqrt n }}\frac{{\left( {2n - 3} \right)!!}}{{\left( {n - 2} \right)!}} \cdot \sum\limits_{k = 0}^{n - 2} {\frac{{{{\left( { - 1} \right)}^k}}}{{k!}}}  = \frac{2}{{e\sqrt \pi  }}.\]

涉及到星形线的一个运动轨迹问题

前几天在群里被问到一个定长木棍在竖直墙边上运动的轨迹问题,还是挺有趣的,涉及到星形线,解决起来也不简单.

(K神问题)长度为$L$的木棍从竖直位置开始沿墙壁一直滑到地面(墙壁与地面垂直),则木棍扫过的面积为?
解.不妨设木棍与竖直墙面的交点为$A(0,L\sin\theta)$,与水平墙面的交点为$B(L\cos\theta,0)$,则我们有棍上任意一点的坐标$(x,y)$满足
\[\left\{ \begin{array}{l}x = \alpha L\cos \theta \\y = \left( {1 - \alpha } \right)L\sin \theta\end{array} \right. \Rightarrow \frac{{{x^2}}}{{{\alpha ^2}{L^2}}} + \frac{{{y^2}}}{{{{\left( {1 - \alpha } \right)}^2}{L^2}}} = 1.\]
由此得
\[ \Rightarrow y = \sqrt {{{\left( {1 - \alpha } \right)}^2}{L^2} - {x^2}{{\left( {\frac{{1 - \alpha }}{\alpha }} \right)}^2}} ,0 < \alpha  < 1,x,L > 0.\]
我们有
\begin{align*}y &= \sqrt {{{\left( {1 - \alpha } \right)}^2}{L^2} - {x^2}{{\left( {\frac{{1 - \alpha }}{\alpha }} \right)}^2}}  = \sqrt {{{\left( {1 - \alpha } \right)}^2}{L^2} - {x^2}{{\left( {\frac{1}{\alpha } - 1} \right)}^2}} \\&= \sqrt {{L^2} - {x^2} + {L^2}{\alpha ^2} - 2{L^2}\alpha  - \frac{{{x^2}}}{{{\alpha ^2}}} + 2\frac{{{x^2}}}{\alpha }}.\end{align*}
令\[g\left( \alpha  \right) = {L^2} - {x^2} + {L^2}{\alpha ^2} - 2{L^2}\alpha  - \frac{{{x^2}}}{{{\alpha ^2}}} + 2\frac{{{x^2}}}{\alpha },\]则\[g'\left( \alpha  \right) = 2{L^2}\alpha  - 2{L^2} + 2\frac{{{x^2}}}{{{\alpha ^3}}} - 2\frac{{{x^2}}}{{{\alpha ^2}}} = \frac{2}{{{\alpha ^3}}}\left( {{L^2}{\alpha ^4} - {L^2}{\alpha ^3} - {x^2}\alpha  + {x^2}} \right) = \frac{{2\left( {1 - \alpha } \right)\left( {{x^2} - {L^2}{\alpha ^3}} \right)}}{{{\alpha ^3}}}.\] 因此
\begin{align*}g\left( \alpha  \right) &\le g\left( {{{\left( {\frac{x}{L}} \right)}^{\frac{2}{3}}}} \right) = {L^2} - {x^2} + {L^2}{\left( {\frac{x}{L}} \right)^{\frac{4}{3}}} - 2{L^2}{\left( {\frac{x}{L}} \right)^{\frac{2}{3}}} - {x^2}{\left( {\frac{L}{x}} \right)^{\frac{4}{3}}} + 2{x^2}{\left( {\frac{L}{x}} \right)^{\frac{2}{3}}}\\&= {L^2} - {x^2} + 3{L^{\frac{2}{3}}}{x^{\frac{4}{3}}} - 3{L^{\frac{4}{3}}}{x^{\frac{2}{3}}}.\end{align*}
故我们有木棍扫过的区域为\[0 \le y \le {L^2} - {x^2} + 3{L^{\frac{2}{3}}}{x^{\frac{4}{3}}} - 3{L^{\frac{4}{3}}}{x^{\frac{2}{3}}},x \in \left[ {0,L} \right].\]
故所求面积为
\[\int_0^L {\sqrt {{L^2} - {x^2} + 3{L^{\frac{2}{3}}}{x^{\frac{4}{3}}} - 3{L^{\frac{4}{3}}}{x^{\frac{2}{3}}}} dx} \underline{\underline {{\text{令}u = \frac{x}{L}}}} {L^2}\int_0^1 {\sqrt {1 - {u^2} + 3{u^{\frac{4}{3}}} - 3{u^{\frac{2}{3}}}} du}  = \frac{{3\pi }}{{32}}{L^2}.\]
事实上
\begin{align*}\int_0^1 {\sqrt {1 - {u^2} + 3{u^{\frac{4}{3}}} - 3{u^{\frac{2}{3}}}} du}  &= \int_0^1 {\sqrt {{{\left( {1 - {u^{\frac{2}{3}}}} \right)}^3}} du} \underline{\underline {{\text{令}u = {{\cos }^3}v}}} 3\int_0^{\frac{\pi }{2}} {{{\sin }^4}v{{\cos }^2}vdv} \\&= 3\int_0^{\frac{\pi }{2}} {{{\sin }^4}vdv}  - 3\int_0^{\frac{\pi }{2}} {{{\sin }^6}vdv}  = \frac{{3\pi }}{{32}}.\end{align*}

十大坑爹高数题(转自哆嗒数学网)

开学啦!进了大学,数学上,第一个要学习的就是高等数学或者数学分析了。这是让有的人感到头痛,有的人带到新奇的学科。利用高中集合和函数做衔接,我们进入了一个新的数学世界。见到之前没见过的函数,之前没见过方法。也颠覆了之前对数学的一些理解。
 
 
 
学习数学,“刷题”是必然不能少。我们不断的做练习,遇到了各种奇怪的题,然后被我们逐个解决。然而,有一些“长”得很像高等数学题目的问题,其实在高数或者数学分析框架下很难解决,而用更高级的办法几乎是秒杀。高数高手们遇到了这些问题,就算是掉坑里了。
 
 
 
这里,哆嗒数学网为你列举10个“坑”。问题方式为了方便大家参与讨论,都以“是否”形式提问。以下提到的函数,如无特殊说明,都为$R\to R$的实函数。
 
 
 
 
 
No.10
 
函数的单调性高中就学啦,一个单调函数可以是$y=x$这样的连续函数,还可以是$y=x\dot {\rm sgn(x)}$这样有间断点的函数。你还能写出很多单调函数,有无穷多个间断点。不过你画图象时,这些函数的间断点大概都是一个一个离散开的。那么间断点可能稠密吗?
 
问题:是否存在一个在无理数点连续,有理数点不连续的严格单调函数。
 
高级秒杀:有理数是可数的。利用级数构造一个特别的函数。见http://duodaa.com/?qa=906,第二题。
 
 
 
No.9
 
有的人也许学了黎曼函数$R(x)$,他是一个有所有无理点处连续,所有有理点处不连续的函数。那么可以反过来吗?
 
问题:是否存在一个在有理数点连续,在无理数点不连续的函数。
 
高级秒杀:先要知道$G\delta$集,$F\delta$集的概念,连续点集只能是$G\delta$.而有理数集不是$G\delta$集。
 
 
 
No.8
 
好了,我们学了导数了,虽然高中也学过,没有像高数那样,平凡的对一个函数某点是否可导进行讨论。“可导必然连续,连续不必可导”,这一句顺口溜一直记于脑海。老师还说,存在处处连续,但处处不可导的函数呢。那么,对于严格单调函数,会怎么样呢。
 
问题:是否存在一个严格单调,但处处不可导的函数。
 
高级秒杀:有个定理是说:单调函数是几乎处处可导的。
 
 
 
No.7
 
一个可导函数$f(x)$求导数后变成了$f'(x)$,$f'(x)$还是一个关于x的函数呢。$f'(x)$可能不再连续呢!那么$f'(x)$可能处处不连续吗?
 
问题:是否存在一个可导函数,它地导函数处处不连续。
 
高级秒杀:连续函数列只能收敛到一个间断点集为第一纲集的函数。而实数集是第二纲集。
 
 
 
No.6
 
函数函足$f(x+y)=f(x)+f(y)$,高中就见过啦。高中还让你证明他是奇函数,在多给一些已知条件的情况下,求$f(1)$、$f(8)$什么的。到了大学,在给定$f(x)$连续情况下,我们能证明f(x)的图象一定是过原点的直线,那么如果$f(x)$不连续呢。
 
问题:如果函数函足$f(x+y)=f(x)+f(y)$,$f(x)$是否有不连续的例子。
 
高级秒杀:实数看成有理数域上的线性空间是无限维的,还要用到线性代数中基的概念。当然,我们承认选择公理。见http://www.duodaa.com/?qa=704.
 
 
 
No.5
 
高中里的集合交并运算,都是有限个里在做,多没意思。大学里可以对无限个集合求交并啦。比如$R$可以写成形如$[n,n+1)$的并集,其中$n$跑遍整数集合。注意到,对于不同的整数,他们还两两不交呢。那么把半开半闭区间改成闭区间呢?
 
问题:实数集是否能写成一列不相交闭区间的并。
 
高级秒杀需要知道:基数、完备集的概念,完备集的基数是不可数的。而如果可以写成,那些区间的端点可以构成完备集。
 
 
 
No.4
 
泰勒展开真神奇,能把一些函数写成一个幂级数的形式。但我们一定也知道了,就算是一个无穷次可导的函数,他本身也不一定等于它的泰勒级数。那么展开式是多项式的情况呢?
 
问题:一个无穷次可导函数在任意一点的泰勒展开式都是多项式,这个函数是否是多项式。
 
高级秒杀:利用贝尔纲定理,精巧的构造一些东西,大概不能算秒杀。见http://www.duodaa.com/?qa=920
 
 
 
No.3
 
对于形如两个数列取幂${f(n)}^{g(n)}$这样的,计算极限,我们有了很多办法。比如凑重要极限形式计算,取对数计算等等。但有一些形式非常简单的极限,解决却不容易。
 
问题:$n\to\infty$时,数列${|\sin(n)|}^\frac 1n$的极限是否等于1。
 
高级秒杀:刘维尔数的概念,以及$\pi$不是刘维尔数。后者是Mahler在1953年的论文上写的,不过,如果不是专门这个方向的一般看不懂,哆嗒数学网的小编也看不懂。
 
http://www.duodaa.com/?qa=2476/
 
 
 
No.2
 
高中就知道了自然对数底e,老师还说他是无理数,但没告诉我为什么是无理数。上了大学,我们终学会了如何证明e是无理数。于是,跃跃欲试,要证明其他数是否是无理数了。
 
问题:根号2的根号2次方是否是有理数。
 
高级秒杀:格尔丰德-施奈德定理可以推出他是超越数,当然就是无理数啦。
 
 
 
No.1
 
接上个问题。同样,我们还学会了证明圆周率π是无理数。两个无理数相加可不一定是无理数呢。
 
问题:$e+\pi$是否是有理数。
 
高级秒杀:些问题人类还没有解决。你能秒杀我叫你大神!。
 
另见:http://tieba.baidu.com/p/3267316241?pn=1

一系列类似积分相等的证明

前阵子四叶群里有人问道下面这题,虽感觉此题结论优美但无从下手.

(1)证明:\[\frac{1}{{\sqrt {2\pi } }}\int_z^\infty  {{e^{ - \frac{1}{2}{x^2}}}dx}  = \frac{1}{\pi }\int_0^{\frac{\pi }{2}} {{e^{ - \frac{{{z^2}}}{{2{{\sin }^2}x}}}}dx} \]
(2)证明:\[{\left( {\frac{1}{{\sqrt {2\pi } }}\int_z^\infty  {{e^{ - \frac{1}{2}{x^2}}}dx} } \right)^2} = \frac{1}{\pi }\int_0^{\frac{\pi }{4}} {{e^{ - \frac{{{z^2}}}{{2{{\sin }^2}x}}}}dx} .\]
(3)当$n>2$时,
\[{\left( {\frac{1}{{\sqrt {2\pi } }}\int_z^\infty  {{e^{ - \frac{1}{2}{x^2}}}dx} } \right)^n} = \frac{1}{\pi }\int_0^{\frac{\pi }{{2n}}} {{e^{ - \frac{{{z^2}}}{{2{{\sin }^2}x}}}}dx}\]
是否成立?

一个很火的积分题

求解\[\displaystyle \int_0^1 \frac{\log^2(1-x)\log(x)}{x}dx=-\frac{\pi^4}{180}.\]

解.$For $|z|<1$ we have that

\[S=\sum\limits_{j=1}^{+\infty }{H_{j}z^{j}}=-\frac{\ln \left( 1-z \right)}{1-z}.\]
Expanding the logarithm and the geometric series
\begin{align*}S&=-\frac{\ln \left( 1-z \right)}{1-z}\\&=\frac{1}{1-z}\sum\limits_{j=1}^{+\infty }{\frac{z^{j}}{j}}=\frac{1}{1-z}\left( z+\frac{z^{2}}{2}+\frac{z^{3}}{3}+\frac{z^{4}}{4}+... \right)=\left( 1+z+z^{2}+... \right)\left( z+\frac{z^{2}}{2}+\frac{z^{3}}{3}+\frac{z^{4}}{4}+... \right)\\&=\left( z+\frac{z^{2}}{2}+\frac{z^{3}}{3}+\frac{z^{4}}{4}+... \right)+z\left( z+\frac{z^{2}}{2}+\frac{z^{3}}{3}+\frac{z^{4}}{4}+... \right)+z^{2}\left( z+\frac{z^{2}}{2}+\frac{z^{3}}{3}+\frac{z^{4}}{4}+... \right)+...\\&=\left( z+\frac{z^{2}}{2}+\frac{z^{3}}{3}+\frac{z^{4}}{4}+... \right)+\left( z^{2}+\frac{z^{3}}{2}+\frac{z^{4}}{3}+\frac{z^{5}}{4}+... \right)+\left( z^{3}+\frac{z^{4}}{2}+\frac{z^{5}}{3}+\frac{z^{6}}{4}+.. \right)+...\\&=z+\left( 1+\frac{1}{2} \right)z^{2}+\left( 1+\frac{1}{2}+\frac{1}{3} \right)z^{3}+\left( 1+\frac{1}{2}+\frac{1}{3}+\frac{1}{4} \right)z^{4}+\left( 1+\frac{1}{2}+\frac{1}{3}+\frac{1}{4}+\frac{1}{5} \right)z^{5}+...\\&=H_{1}z+H_{2}z^{2}+H_{3}z^{3}+H_{4}z^{4}+H_{5}z^{5}+...=\sum\limits_{j=1}^{+\infty }{H_{j}z^{j}}.\end{align*}
Aso we know that
\[H_{n}=1+\frac{1}{2}+...+\frac{1}{n}=\int\limits_{0}^{1}{\left( 1+x+...+x^{n-1} \right)dx}=\int\limits_{0}^{1}{\frac{1-x^{n}}{1-x}dx}\]
with $\displaystyle H_0=0$ by convention. Consider the polylogarithm function,We know that
\[\frac{d}{dx}Li_{n}\left( x \right)=\frac{Li_{n-1}\left( x \right)}{x}.\]
Then
\begin{align*}I&=\int\limits_{0}^{1}{\ln x\ln ^{2}\left( 1-x \right)\frac{dx}{x}}=\int\limits_{0}^{1}{\frac{\ln \left( 1-x \right)}{1-x}\ln ^{2}xdx}=-\int\limits_{0}^{1}{\sum\limits_{j=1}^{+\infty }{H_{j}x^{j}}\ln ^{2}xdx}\\&=-\sum\limits_{j=1}^{+\infty }{H_{j}\int\limits_{0}^{1}{x^{j}\ln ^{2}xdx}}=-2\sum\limits_{j=1}^{+\infty }{\frac{H_{j}}{\left( j+1 \right)^{3}}}\end{align*}
as
\[K\left( j \right)=\int\limits_{0}^{1}{x^{j}dx}=\left( \frac{x^{j+1}}{j+1} \right)\left| _{0}^{1} \right.=\frac{1}{j+1}\Rightarrow K''\left( j \right)=\int\limits_{0}^{1}{x^{j}\ln ^{2}xdx}=-\frac{1}{\left( j+1 \right)^{2}}=\frac{2}{\left( j+1 \right)^{3}}.\]
Denote $S$ the sum$\displaystyle S=\sum\limits_{j=1}^{+\infty }{\frac{H_{j}}{\left( j+1 \right)^{3}}}.$
\begin{align*}S&=\sum\limits_{j=1}^{+\infty }{\frac{H_{j}}{\left( j+1 \right)^{3}}}=\sum\limits_{j=1}^{+\infty }{\left( \frac{\frac{1}{j+1}-\frac{1}{j+1}+H_{j}}{\left( j+1 \right)^{3}} \right)}\\&=\sum\limits_{j=1}^{+\infty }{\left( \frac{H_{j+1}-\frac{1}{j+1}}{\left( j+1 \right)^{3}} \right)}=\sum\limits_{j=1}^{+\infty }{\left( \frac{H_{j+1}}{\left( j+1 \right)^{3}}-\frac{1}{\left( j+1 \right)^{4}} \right)}\\&=\sum\limits_{j=0}^{+\infty }{\left( \frac{H_{j+1}}{\left( j+1 \right)^{3}}-\frac{1}{\left( j+1 \right)^{4}} \right)}=\sum\limits_{j=1}^{+\infty }{\left( \frac{H_{j}}{j^{3}}-\frac{1}{j^{4}} \right)}=-\zeta \left( 4 \right)+\sum\limits_{j=1}^{+\infty }{\frac{H_{j}}{j^{3}}}\\&=-\zeta \left( 4 \right)+\sum\limits_{j=1}^{+\infty }{\left( \int\limits_{0}^{1}{\frac{1-x^{j}}{1-x}}\frac{1}{j^{3}}dx \right)}=-\zeta \left( 4 \right)+\int\limits_{0}^{1}{\frac{1}{1-x}\sum\limits_{j=1}^{+\infty }{\left( \frac{1-x^{j}}{j^{3}} \right)}}dx\\&=-\zeta \left( 4 \right)+\int\limits_{0}^{1}{\frac{1}{1-x}\left( \zeta \left( 3 \right)-\sum\limits_{j=1}^{+\infty }{\frac{x^{j}}{j^{3}}} \right)}dx=-\zeta \left( 4 \right)+\int\limits_{0}^{1}{\frac{1}{1-x}\left( \zeta \left( 3 \right)-Li_{3}\left( x \right) \right)}dx.\end{align*}
Using integration by parts
Let $\displaystyle u=\zeta \left( 3 \right)-Li_{3}\left( x \right)$ and $\displaystyle dv=\frac{dx}{1?x}$.Then$\displaystyle du=-\frac{Li_{2}\left( x \right)}{x}$and$\displaystyle v=-\log \left| 1-x \right|$.So, the sum is equal to
\begin{align*}S&=-\zeta \left( 4 \right)+\left( Li_{3}\left( x \right)-\zeta \left( 3 \right) \right)\log \left| 1-x \right|\left| _{0}^{1} \right.-\int\limits_{0}^{1}{\log \left( 1-x \right)\frac{Li_{2}\left( x \right)}{x}}dx\\&=-\zeta \left( 4 \right)-\int\limits_{0}^{1}{Li_{2}\left( x \right)\frac{\log \left( 1-x \right)}{x}}dx.\end{align*}
Making the following change of variable
\begin{align*}u&=Li_{2}\left( x \right)\Rightarrow du=-\frac{\log \left( 1-x \right)}{x}dx\\S&=-\zeta \left( 4 \right)+\int\limits_{0}^{1}{Li_{2}\left( x \right)\left( Li_{2}\left( x \right) \right)^{'}}dx\\&=-\zeta \left( 4 \right)+\frac{1}{2}\left( Li_{2}^{2}\left( x \right) \right)\left| _{0}^{1} \right.=-\zeta \left( 4 \right)+\frac{1}{2}\left( Li_{2}^{2}\left( 1 \right)-Li_{2}^{2}\left( 0 \right) \right).\end{align*}
where $\displaystyle Li_{2}\left( x \right)=\sum\limits_{j=1}^{+\infty }{\frac{x^{j}}{j^{2}}}.$Then
\[S=-\zeta \left( 4 \right)+\frac{1}{2}\zeta ^{2}\left( 2 \right)=\frac{\pi ^{4}}{72}-\frac{\pi ^{4}}{90}=\frac{\pi ^{4}}{18}\left( \frac{1}{4}-\frac{1}{5} \right)=\frac{\pi ^{4}}{18}\left( \frac{1}{20} \right)=\frac{\pi ^{4}}{360}.\]
Finally we conclude
\[I=-2\sum\limits_{j=1}^{+\infty }{\frac{H_{j}}{\left( j+1 \right)^{3}}}=-2\left( -\zeta \left( 4 \right)+\frac{1}{2}\zeta ^{2}\left( 2 \right) \right)=-2\cdot \frac{\pi ^{4}}{360}=-\frac{\pi ^{4}}{180}.\]

一个与多项式分拆有关的级数题

求证:对于$\forall k\in N_+$,必有\[\sum\limits_{n = 1}^\infty  {\frac{{{n^k}}}{{n!}}}\]是$e$的整数倍.
证明.先证明一个引理:对$\forall k\in N_+$,均有${n^k} = {a_0}n + {a_1}n\left( {n - 1} \right) + {a_2}n\left( {n - 1} \right)\left( {n - 2} \right) +  \cdots  + {a_k}n\left( {n - 1} \right)\left( {n - 2} \right) \cdots \left( {n - k+1} \right)$成立,其中$a_1+a_2+\ldots+a_i+\ldots+a_{k-1}(i=0,1,2,\ldots,k)$为整数(事实上,$a_i$均为正整数,$i\in N^+$).
证明.写得规范点,我们有:\[{x^k} = {a_0}x + {a_1}x\left( {x - 1} \right) + {a_2}x\left( {x - 1} \right)\left( {x - 2} \right) +  \cdots  + {a_{k-1}}x\left( {x - 1} \right)\left( {x - 2} \right) \cdots \left( {x - k+1} \right).\]
比较两边$x^k$的系数得知$a_{k-1}=1$,即
\begin{align*}&x\left( {x - 1} \right)\left( {x - 2} \right) \cdots \left( {x - k + 1} \right) = {x^k} - {a_{k - 2}}x\left( {x - 1} \right)\left( {x - 2} \right) \cdots \left( {x - k + 2} \right) \\&-  \cdots  - {a_2}x\left( {x - 1} \right)\left( {x - 2} \right) - {a_1}x\left( {x - 1} \right) - {a_0}x.\end{align*}
 
再比较两边$x^{k-1}$的系数得$a_{k-2}=\frac{k(k-1)}{2}$,由此
\[\frac{{{{\left( {1 + 2 +  \cdots  + \left( {k - 1} \right)} \right)}^2} - \left( {{1^2} + {2^2} +  \cdots  + {{\left( {k - 1} \right)}^2}} \right)}}{2} = \left( {1 + 2 +  \cdots  + \left( {k - 2} \right)} \right){a_{k - 2}} - {a_{k - 3}}.\]故\[{a_{k - 3}} = \frac{{\left( {3k - 5} \right)\left( {k - 2} \right)\left( {k - 1} \right)k}}{{24}}.\]
一般地,考察等式两边$x^n$的系数我们有
\begin{align*}&{\left( { - 1} \right)^{k - n}}\sum\limits_{1 \le {k_1} < {k_2} <  \cdots  < {k_{k - n}} \le k - 1} {{k_1}{k_2} \cdots {k_{k - n}}}  = {\left( { - 1} \right)^{k - n}}{a_{k - 2}}\sum\limits_{1 \le {k_1} < {k_2} <  \cdots  < {k_{k - n - 1}} \le k - 2} {{k_1}{k_2} \cdots {k_{k - n - 1}}} \\&+ {\left( { - 1} \right)^{k - n - 1}}{a_{k - 3}}\sum\limits_{1 \le {k_1} < {k_2} <  \cdots  < {k_{k - n - 2}} \le k - 3} {{k_1}{k_2} \cdots {k_{k - n - 2}}}  +  \cdots  + {\left( { - 1} \right)^2}{a_n}\sum\limits_{1 \le {k_1} \le n} {{k_1}} {x^n} + {\left( { - 1} \right)^1}{a_{n - 1}}.\end{align*}
依此递推式得$a_0=1.$且$a_i,i=0,1,\ldots,k-2$均为正整数.
回到原题,我们有
\begin{align*}\sum\limits_{n = 1}^\infty  {\frac{{{n^k}}}{{n!}}}  &= \sum\limits_{n = 1}^\infty  {\frac{{{a_0}n + {a_1}n\left( {n - 1} \right) + {a_2}n\left( {n - 1} \right)\left( {n - 2} \right) +  \cdots  + {a_{k - 1}}n\left( {n - 1} \right)\left( {n - 2} \right) \cdots \left( {n - k + 1} \right)}}{{n!}}} \\&= {a_0}\sum\limits_{n = 1}^\infty  {\frac{1}{{\left( {n - 1} \right)!}}}  + {a_1}\sum\limits_{n = 2}^\infty  {\frac{1}{{\left( {n - 2} \right)!}}}  + {a_2}\sum\limits_{n = 3}^\infty  {\frac{1}{{\left( {n - 3} \right)!}}}  +  \cdots  + {a_{k - 1}}\sum\limits_{n = k}^\infty  {\frac{1}{{\left( {n - k} \right)!}}} \\&= \left( {{a_0} + {a_1} + {a_2} +  \cdots {a_{k - 1}}} \right)e = Ne.\end{align*}